Intégrale double généralisée

2

Réponses

  • Il y a ce document qui faisait référence mais qui n'a pas été mis à jour, dans le contenu, depuis plus de dix ans,

    http://empslocal.ex.ac.uk/people/staff/rjchapma/etc/zeta2.pdf

    et on peut consulter,

    https://math.stackexchange.com/questions/8337/different-methods-to-compute-sum-limits-k-1-infty-frac1k2/1688540
  • Eric:

    Si tu pouvais obtenir une copie de cet article, ce serait magnifique.
  • J'ai prévu de passer à la BU de Jussieu la semaine prochaine. Quant à faire une copie de l'article ... je vais emmener mon appareil photo.

    Et sinon, quelqu'un a cité la démonstration utilisant le théorème de Tannery ?
  • Le livre de Leveque est disponible ici:
    https://ia601900.us.archive.org/34/items/in.ernet.dli.2015.134692/2015.134692.Topics-In-Number-Theory-Volume-1.pdf

    Dans cette copie, je ne vois pas de rajout au delà de 1965.

    Eric:

    Ce dont tu parles m'évoque quelque chose mais je ne me souviens plus où j'ai lu ça.

    Probablement que j'avais lu ça ici:
    https://arxiv.org/pdf/1301.3659.pdf
  • Ca a été publié dans le Monthly en 2002 :
  • Ah, je connais ce nom. Je dois l'avoir dans mon fouillis . Merci pour ce rappel.
  • Bonjour.
    Si je n'ai pas rêvé, un message donnait le moyen de télécharger le livre de LeVeque, mais il a disparu subrepticement, et et je ne l'ai pas utilisé à temps. Si quelqu'un peut me le communiquer par message privé, je le remercierai vivement et en échange je lui ferai cadeau d'une information précieuse.
    Bonne journée.
    Fr. Ch.
  • Oups mille excuses, le lien est encore présent. Chat échaudé craint l'eau froide.
  • N'écoutant que mon courage j'ai bravé la canicule et je suis allé à la Bibliothèque Math-Recherche de Jussieu, et j'ai trouvé l'article de Goldscheider de 1913, ce qui établit, me semble-t-il, l'origine de la démonstration au moyen de l'intégrale double.
    Cette bibliothèque est vraiment très riche en revues, d'un grand nombre de pays et d'époques. J'ai bien fait d'y aller sans attendre car elle ferme demain pour n'ouvrir que le 21 août, et d'y aller le matin car elle ferme en juillet à 17 h 30.
    Bonne soirée.
    Fr. Ch.
    20/07/2017
  • Merci Chaurien pour ces documents.
  • Je propose de montrer, sans connaître la valeur de $\zeta(2)$, que:

    $\displaystyle J:=\int_0^1 \frac{\ln(1+x)}{x}dx=\frac{\pi^2}{12}$

    Pour ce faire on définit sur $[0;1]$ la fonction $f$ par:

    $\displaystyle f(s)=\int_0^{\frac{\pi}{2}} \arctan\left(\frac{\sin t}{\cos t+s}\right)\,dt$

    On remarque que,

    $\begin{align} f(0)&=\int_0^{\frac{\pi}{2}}\arctan\left(\frac{\sin t}{\cos t}\right)\,dt\\
    &=\int_0^{\frac{\pi}{2}} t\,dt\\
    &=\left[\frac{t^2}{2}\right]_0^{\frac{\pi}{2}}\\
    &=\frac{\pi^2}{8}
    \end{align}$

    Pour tout $t$ dans $\left[0,\frac{\pi}{2}\right]$,

    $\begin{align} \frac{\sin t}{\cos t+1}&=\frac{2\sin\left(\frac{t}{2}\right)\cos\left(\frac{t}{2}\right)}{\cos^2\left(\frac{t}{2}\right)-\sin^2\left(\frac{t}{2}\right)+1}\\
    &=\frac{2\sin\left(\frac{t}{2}\right)\cos\left(\frac{t}{2}\right)}{2\cos^2\left(\frac{t}{2}\right)}\\
    &=\tan\left(\frac{t}{2}\right)
    \end{align}$

    Donc,

    $\begin{align} f(1)&=\int_0^{\frac{\pi}{2}}\arctan\left(\frac{\sin t}{\cos t+1}\right)\,dt\\
    &=\int_0^{\frac{\pi}{2}}\arctan\left(\tan\left(\frac{t}{2}\right)\right)\,dt\\
    &=\int_0^{\frac{\pi}{2}} \frac{t}{2}\,dt\\
    &=\left[\frac{t^2}{4}\right]_0^{\frac{\pi}{2}}\\
    &=\frac{\pi^2}{16}
    \end{align}$

    Pour tout $s$ de $[0,1]$,

    $\begin{align}
    f^\prime(s)&=-\int_0^{\frac{\pi}{2}}\frac{\sin t}{1+2s\cos t+s^2}\,dt\\
    &=\left[\frac{\ln(1+2s\cos t+s^2)}{2s}\right]_0^{\frac{\pi}{2}}\\
    &=\frac{1}{2}\frac{\ln\left(1+s^2\right)}{s}-\frac{\ln\left(1+s\right)}{s}
    \end{align}$

    Ainsi,

    $\begin{align}
    f(1)-f(0)&=\int_0^1 f^\prime(s)ds\\
    &=\frac{1}{2}\int_0^1\frac{\ln\left(1+s^2\right)}{s}\,ds-\int_0^1 \frac{\ln\left(1+s\right)}{s}\,ds\\
    \end{align}$

    Dans la première intégrale on fait le changement de variable $y=s^2$, donc,

    $\displaystyle f(1)-f(0)=-\frac{3}{4}J$

    Or,

    $\begin{align} f(1)-f(0)&=\frac{\pi^2}{16}-\frac{\pi^2}{8}\\
    &=-\frac{\pi^2}{16}
    \end{align}$

    Donc,

    $\boxed{\displaystyle J=\frac{\pi^2}{12}}$


    PS:
    J'étais en train de calculer une intégrale et j'ai pensé à cette façon de calculer J.
    J'imagine que je suis influencé par une preuve que j'ai lue quelque part mais je n'ai pas encore retrouvé la source (si elle existe).
  • Merci FDP, moi aussi je cherchais des méthodes pour calculer $\int_0^1 \frac {\ln(x)}{x+1 }dx$
    Il semble que ton intégrale est l'opposé de celui ci
    Le 😄 Farceur


  • A partir de la connaissance de la valeur de $\zeta(2)$ ce n'est pas difficile d'obtenir la valeur de,
    $\displaystyle \int_0^1 \frac{\ln(1+t)}{t}dt$.

    $\begin{align}
    \int_0^1 \frac{\ln(1+t)}{t}\,dt+\int_0^1 \frac{\ln(1-t)}{t}\,dt=\int_0^1 \frac{\ln(1-t^2)}{t}\,dt
    \end{align}$

    Et on fait le changement de variable $y=t^2$ dans l'intégrale du membre de droite,

    Donc,
    $\begin{align}
    \int_0^1 \frac{\ln(1+t)}{t}\,dt+\int_0^1 \frac{\ln(1-t)}{t}\,dt=\frac{1}{2}\int_0^1 \frac{\ln(1-t)}{t}\,dt
    \end{align}$

    On a donc,

    $\begin{align}
    \int_0^1 \frac{\ln(1+t)}{t}\,dt=-\frac{1}{2}\int_0^1 \frac{\ln(1-t)}{t}\,dt
    \end{align}$


    Et on peut développer en série l'intégrande de cette dernière intégrale et intégrer la série entière obtenue terme à terme pour obtenir qu'elle vaut $-\zeta(2)$

    Et donc,

    $\boxed{\displaystyle \int_0^1 \frac{\ln(1+t)}{t}\,dt=\frac{1}{2}\zeta(2)}$
  • Peut-on montrer que la fonction f(x,y) proposée est intégrable en en donnant un équivalent "à 2 variables" en (1;1) ?
  • Totem:

    Je n'ai pas compris ta question.
    Par ailleurs, tu parles de l'intégrande de l'intégrale du tout premier message?
  • Oui en effet l'intégrande. J'ai essayé de faire un DL mais avec une fonction de 2 variables je m'emmêle...!
    Si on arrive à montrer que l'intégrande est équivalent à quelque chose d'intégrable en (1;1) , c'est chouette :-) sinon comment peut-on "prévoir" que l'intégrale double ne diverge pas en (1;1) ?
  • Bonjour,

    La démonstration la plus simple n'est-elle pas celle-ci ?

    $\displaystyle \forall x \in \R, \sin (\pi x) = \pi x (1-x^2) (1-{x^2 \over 4})(1-{x^2 \over 9}) \cdots = \pi x + \pi x^3 (1+\frac14 + \frac19 + \cdots) + \pi x^5 ({1 \over 1.4} + {1 \over 1.9} + \cdots + {1 \over 4.9} + \cdots) + \cdots = \\ \displaystyle= \pi x - {\pi^3 \over 6} x^3 + {\pi^5 \over 120} x^5 + \cdots $
    et donc $\displaystyle 1+\frac14 + \frac19 + \cdots = {\pi^2 \over 6}.$
  • Personne ? Peut-être est-il plus pertinent de parler de développement asymptotique...
    Ou trouver une primitive mais je n' y crois guère !
    @YvesM: d'où sort la première ligne ? merci.
  • Bonjour,

    La fonction $\displaystyle z \in \C \mapsto {\sin (\pi z) \over \pi z}$, une fois prolongée par continuité en $0$, a une infinité de zéros, qui sont en $\displaystyle m \in \Z^*.$ La fonction $\displaystyle z \in \C \mapsto \prod_{n \in \N^*} (1-{z^2 \over n^2})$ aussi... elles vallent toutes les deux $1$ en $0$... et un théorème dit qu'elles sont donc égales, lequel ?
  • Si tu multiplies une des fonctions par $\exp(z)$, elle a les mêmes zéros et elle vaut toujours $1$ en $0$. Alors ?
  • Est-ce que je sais moi...d'Alembert-Gauss pour les fonctions méromorphes ? Weierstrass-Hadamard ? c'est wikipédia qui me l' a soufflé hein...
    Notez que je ne sais pas bien ce que c'est...
  • YvesM:

    Le mot "démonstration" est en trop.

    Il faut se rappeler que le pas décisif d'Euler est d'avoir trouvé un moyen d'accélérer la convergence de la série des inverses des carrés non nuls car cette dernière converge très lentement et il faut calculer beaucoup, beaucoup de termes pour avoir une valeur d'une ou deux décimales après la virgule (à la main c'est rédhibitoire)

    Dès qu'Euler a obtenu une valeur approchée suffisamment précise de la série susmentionnée il a été capable de deviner comment numériquement on l'obtenait et cela a du lui donner l'idée de cette "démonstration" rappelée par YvesM et qui n'en était déjà pas une même à l'époque d'Euler. Ce dernier a du revoir sa copie et il a fourni plus tard, à ma connaissance, deux preuves correctes de ce résultat au regard des standards actuels de rigueur dans une démonstration.
  • SVP comment passe-t-on de Zeta (2) alternée à Zeta (2) ? J'ai coupé la somme de Zeta (2) alternée en 2 : d'un côté les n pairs, d'un autre les n impairs. Cela permet d'exhiber Zeta (2) Mais après je suis bloqué car j'ignore la somme des 1/(2n+1)² ...qui doit dépendre elle même de Zeta (2) je pense !

    Merci.
  • $$ \zeta(s) = \sum \frac{1}{(2n+1)^s} + \sum \frac{1}{(2n)^s} = \sum \frac{1}{(2n+1)^s} + {1 \over 2^s} \zeta(s)$$
  • Sinon avec le produit
    $$
    \zeta(s) = \prod_p \left( 1 - {1 \over p^s }\right)^{-1} = \left( 1 - {1 \over 2^s }\right)^{-1} \prod_{p> 2} \left( 1 - {1 \over p^s }\right)^{-1} = \left( 1 - {1 \over 2^s }\right)^{-1} \times \sum \frac{1}{(2n+1)^s}
    $$
  • Oui mais moi je pars de Zeta(2) alternée et je vais vers Zeta (2)...
  • La série alternée $\beta(2)$ correspondant à $\zeta(2)$ est égale à $-A$ avec $\displaystyle A=\int_0^1 \dfrac{\ln(1+x)}{x}\,dx$

    Si $\displaystyle B=\int_0^1 \frac{\ln(1-x)}{x}\,dx$ alors $\zeta(2)=-B$

    $\displaystyle A+B=\int_0^1 \frac{\ln(1-x^2)}{x}\,dx$ et par le changement de variable $y=x^2$ cette dernière intégrale est égale à $\frac{1}{2}B$

    Donc,

    $\begin{align}\beta(2)&=-A\\&=\frac{1}{2}B\\&=-\frac{1}{2}\zeta(2)\end{align}$
  • Salut j'ai essayé de répondre à

    " Mais après je suis bloqué car j'ignore la somme des 1/(2n+1)² ...qui doit dépendre elle même de Zeta (2) je pense ! "
  • Bonjour,

    Voici une forme du théorème de Weierstrass qui permet d'établir le résultat.

    Soit une fonction entière $f$ qui possède une infinité de zéros $\displaystyle \{a_n : n \in \Z\}$ ; on suppose que $\displaystyle a_0=0$ et que $\displaystyle a_0$ est d'ordre $r \in \N$ ; et de plus on suppose que $\displaystyle a_n \to \infty, (n \to \infty)$ : alors $\displaystyle \forall z \in \C, f(z) = z^r e^{h(z)} \prod_{n \in \Z^*} (1-{z \over a_n}) e^{\sum_{k=1}^{p_r} {1 \over k} ({z\over a_n})^k}$ pour une certaine fonction enitère $h$ et une suite d'entiers positifs $p.$ La suite $p$ est arbitraire pourvu que la série $\displaystyle \sum_{n \in \Z^*} ({z\over a_n})^{p_n+1}$ converge uniformément sur tout compact $\displaystyle K \subset \C.$

    On applique ce théorème à la fonction $\displaystyle f: z \in \C \mapsto \sin z.$ On a alors $\displaystyle a_n = \pi n, n \in \Z$, $\displaystyle r=1$ et puisque la série $\displaystyle \sum_{n \in \Z^*} ({z \over \pi n})^2 = 2 \sum_{n \geq 1} ({z \over \pi n})^2 $ converge uniformément sur tout compact $\displaystyle K \subset \C$, on choisit la suite $p$ constante et égale à $1.$
    On a donc $\displaystyle \forall z \in \C, \sin z = z e^{h(z)} \prod_{n \in \Z^*} (1-{z \over \pi n}) e^{{z \over \pi n}}.$
    On considère un compact $K$ qui ne contient pas de zéros de cette fonction $f$ et alors, $\displaystyle \forall z \in K, \ln \sin z = h(z) + \ln z + \sum_{n \in \Z^*} \Big( \ln (1-{z \over \pi n}) + {z \over \pi n}\Big)$, puis $\displaystyle \forall z \in K, {d \over dz} \ln \sin z = {1 \over \tan z} = h'(z) + {1 \over z} + \sum_{n \in \Z^*} ({1 \over z-\pi n} + {1 \over \pi n}).$
    On sait que $\displaystyle \forall z \in K, {1 \over \tan z} = {1 \over z} + \sum_{n \in \Z^*} ({1 \over z-\pi n} + {1 \over \pi n}).$ On en déduit que $\displaystyle \forall z \in K, h'(z)=0.$ Comme le choix du compact est arbitraire, alors $h$ est une fonction constante. On a alors $\displaystyle \forall z \in \C, \sin z = Cz \prod_{n \in \Z^*} (1-{z \over \pi n}) e^{{z \over \pi n}}$ avec $C$ une constante complexe. Puisque $\displaystyle {\sin z \over z} \to 1, (z \to 0)$, alors $\displaystyle C=1$ et, finalement, $\displaystyle \forall z \in \C, \sin z = z \prod_{n \in \Z^*} (1-{z \over \pi n}) e^{{z \over \pi n}}.$
    On manipule cette expression pour aboutir au résultat : $\displaystyle \forall z \in \C, \sin z = z \lim_{N \to +\infty}\prod_{n =-N, n \neq 0}^{N} (1-{z \over \pi n}) e^{{z \over \pi n}} = z\lim_{N \to +\infty}\prod_{n =1}^{N} (1-{z^2 \over \pi^2 n^2}) = z \prod_{n \geq 1} (1-{z^2 \over \pi^2 n^2}) .$
  • @ tous :merci !
    Ok, je ne pensais pas qu'il fallait repasser par des intégrales pour passer de Beta(2) à Zeta(2), je pensais qu'on pouvait s'en sortir juste en manipulant des séries...mais non !:-)
  • Totem:

    On peut faire le calcul aussi directement sur les séries. Il faut utiliser pour terminer le calcul de Flipflop:
    http://www.les-mathematiques.net/phorum/read.php?4,1400462,1510816#msg-1510816
  • Oui mais dans ce cas on a besoin de Zeta(2) pour calculer Zeta(2) ...ça se mord la queue ! 8-)
  • Pas du tout.
    $s>1$,
    $\begin{align}
    \sum_{n=1}^{\infty} \frac{(-1)^n}{n^s}&=\sum_{n \text{ pair non nul}} \frac{(-1)^n}{n^s}+\sum_{n \text{ impair}} \frac{(-1)^n}{n^s}\\
    &=\sum_{n=1}^{\infty} \frac{(-1)^{2n}}{(2n)^s}+\sum_{n=0}^{\infty} \frac{(-1)^{2n+1}}{(2n+1)^s}\\
    &=\sum_{n=1}^{\infty} \frac{1}{(2n)^s}-\sum_{n=0}^{\infty} \frac{1}{(2n+1)^s}\\
    &=\sum_{n=1}^{\infty} \frac{1}{(2n)^s}-\left(\zeta(s)-\sum_{n=1}^{\infty} \frac{1}{(2n)^s}\right)\\
    &=2\sum_{n=1}^{\infty} \frac{1}{(2n)^s}-\zeta(s)\\
    &=\frac{1}{2^{s-1}}\sum_{n=1}^{\infty} \frac{1}{n^s}-\zeta(s)\\
    &=\frac{1}{2^{s-1}}\zeta(s)-\zeta(s)\\
    &=\left(\frac{1}{2^{s-1}}-1\right)\zeta(s)
    \end{align}$
    Puisque un nombre est ou bien pair, ou bien impair, on a :
    $\begin{align}
    \zeta(s)&=\sum_{n=1}^{\infty}\frac{1}{n^s}\\
    &=\sum_{n\text{ pair non nul }}\frac{1}{n^s}+\sum_{n\text{ impair }}\frac{1}{n^s}\\
    &=\sum_{n=1}^{\infty}\frac{1}{(2n)^s}+\sum_{n=0}^{\infty}\frac{1}{(2n+1)^s}
    \end{align}$

    Donc on a bien,
    $\begin{align}
    \sum_{n=0}^{\infty}\frac{1}{(2n+1)^s}=\zeta(s)-\sum_{n=1}^{\infty}\frac{1}{(2n)^s}
    \end{align}$
  • Merci FdP !!!
    J'avais les 3 premières lignes reliant Beta à Zeta, mais bizarrement je n'ai pas pensé à la 4 ème qui est pourtant évidente.
  • Pour obtenir $3$ décimales exactes de $\zeta(2)$ en prenant comme valeurs approchées les termes de la suite (croissante),

    $\displaystyle u_n=\sum_{k=1}^n \frac{1}{k^2}$

    Il faut calculer $u_{1071}$ (c'est à dire additionner $1071$ nombres décimaux) qui vaut $1,6440...$ tandis que $u_{1070}$ vaut $1,6439...$
    (et $\zeta(2)=1,6449...$ )

    En faisant des calculs à la main c'est un travail considérable pour obtenir 3 misérables décimales.

    Comment obtenir autant, voire plus, de décimales en faisant moins de calculs?

    Une manière de faire:

    On montre en intégrant terme à terme le développement en série entière de $\frac{\ln(1-x)}{x}$ que,

    $\displaystyle \sum_{n=1}^{\infty} \frac{1}{2^n n^2}=-\int_0^{\frac{1}{2}}\frac{\ln(1-x)}{x}\,dx$

    Ce nombre a un lien assez simple avec $\zeta(2)$ comme le montre ce qui suit.

    Dans l'intégrale on fait le changement de variable $y=\frac{x}{1-x}$,

    $\begin{align} \int_0^{\frac{1}{2}}\frac{\ln(1-x)}{x}\,dx&=-\int_0^1 \frac{\ln(1+x)}{x(1+x)}\,dx\\
    &=\int_0^1 \left(\frac{\ln(1+x)}{1+x}-\frac{\ln(1+x)}{x}\right)\,dx\\
    &=\left[\frac{1}{2}\ln^2(1+x)\right]_0^1-\int_0^1\frac{\ln(1+x)}{x}\,dx\\
    &=\frac{1}{2}\ln^2 2-\int_0^1\frac{\ln(1+x)}{x}\,dx\\
    \end{align}$

    Et nous savons que

    $\displaystyle \int_0^1\frac{\ln(1+x)}{x}\,dx=\frac{1}{2}\zeta(2)$

    donc,

    $\boxed{\displaystyle \zeta(2)=2\sum_{n=1}^{\infty} \frac{1}{2^n n^2}+\ln^2 2}$


    Pour calculer des valeurs approchées de,
    $\displaystyle \sum_{n=1}^{\infty} \frac{1}{2^n n^2}$

    en utilisant la suite,
    $\displaystyle v_n=\sum_{k=1}^{n} \frac{1}{2^k k^2}$

    Il faut calculer $u_9$ pour obtenir $0,58222...$ (c'est à dire additionner $9$ nombres décimaux).
    A comparer avec,

    $\displaystyle \sum_{n=1}^{\infty} \frac{1}{2^n n^2}=0,58224...$

    PS:
    Les tables de logarithmes existent depuis longtemps.
  • Et on récupère Zeta(2) alternée dans la foulée (:D
  • @Totem : je n'ai pas réussi à obtenir la relation entre la fonction zéta et la fonction zéta alternée en utilisant le produit infini de zéta : $$
    \zeta(s) = \prod_p \left( 1 - {1 \over p^s }\right)^{-1} = \left( 1 - {1 \over 2^s }\right)^{-1} \prod_{p> 2} \left( 1 - {1 \over p^s }\right)^{-1} = \left( 1 - {1 \over 2^s }\right)^{-1} \times \sum \frac{1}{(2n+1)^s}
    $$ On peut remplacer $2$ par $3$ (dans mon calcul $s=2$ pour toi). $$
    \zeta(s) = \prod_p \left( 1 - {1 \over p^s }\right)^{-1} = \left( 1 - {1 \over 3^s }\right)^{-1} \prod_{p \ne 3} \left( 1 - {1 \over p^s }\right)^{-1} = \left( 1 - {1 \over 3^s }\right)^{-1} \times \left( \sum \frac{1}{(3n+1)^s}+\sum \frac{1}{(3n+2)^s} \right)
    $$ Mais je ne sais pas si la série zéta alternée se factorise par la fonction zeta, je pense qu'il faut mettre des caractères dans la machine :-S
  • @FdP
    Ceci ne me semble pas une méthode très pertinente pour une accélération de convergence dans le calcul de $\zeta(2)$ parce qu'il apparaît ce terme $(\ln 2)^2$ qui demande lui aussi à être calculé avec précision.
    Déjà on pourrait proposer une modeste accélération en considérant la série de terme général $\frac {1}{(n(n+1))^2}$, ou quelque chose de similaire.
    Mais si l'on veut mieux, on peut voir du côté du développement en série entière de $ (\arcsin x)^2$.
    Bonne soirée.
    Fr. Ch.
  • Chaurien:

    Une table de logarithmes et une dizaine d'additions et on peut obtenir une précision de 4 chiffres après la virgule de $\zeta(2)$. C'est déjà pas si mal.
  • @ FdP
    Pas d'accord. Tu calcules $\zeta (2) - (\ln 2)^2$ et non $\zeta (2) $ en personne. Ta table de logarithmes est une de ces chères vieilles choses qui ne font plus partie de notre environnement habituel, depuis longtemps, comme le moulin à café à manivelle, et avec raison. Je suis certain que nombre de participants à ce forum n'en ont jamais vu. Et elle ne donne que les logarithmes décimaux.

    Une idée est de calculer ce terme $(\ln 2)^2$ par une autre série entière.
    On peut partir de : $ \displaystyle \frac{-\ln (1-x)}{1-x}=\overset{+\infty }{\underset{n=1}{\sum }}H_{n}x^{n}$, avec : $ \displaystyle H_{n}=\overset{n}{\underset{k=1}{\sum }}\frac{1}{k}$, pour $x \in [-1,1[$.
    Par primitivation : $ \displaystyle (\ln (1-x))^{2}=2\overset{+\infty }{\underset{n=1}{\sum }}\frac{H_{n}}{n+1}x^{n+1}$.

    Pour $x:=-1$, il vient : $ \displaystyle (\ln 2)^{2}=2\overset{+\infty }{\underset{n=1}{\sum }}(-1)^{n+1}\frac{H_{n}}{n+1}$.
    Mais comme tu dis ce n'est pas malin de se positionner à l'extrémité de l'intervalle de convergence, la convergence est bien plus rapide à l'intérieur de celui-ci.

    Pour $x:=\frac{1}{2}$, il vient : $ \displaystyle (\ln 2)^{2}=\overset{+\infty }{\underset{n=1}{\sum }}\frac{H_{n}}{2^{n}(n+1)}$, calcul qui n'est pas très difficile à programmer, et ici encore on aura pas mal de chiffres avec peu de termes.

    J'ai signalé une autre idée dans mon précédent message. J'y reviendrai.

    Bonne journée [*** modéré *** restons dans les mathématiques. AD]
    Fr. Ch.
  • @ FdP
    Je développe mon autre idée.
    On prouve sans mal que : $\displaystyle f(x)=(\arcsin x)^{2}=\underset{n=1}{\overset{+\infty }{\sum }}a_{n}x^{2n}$, avec : $\displaystyle a_{n}=\frac{2.4....(2n-2)}{3.5....(2n-1)}\cdot \frac{1}{n}\sim \frac{\sqrt{\pi }}{2n^{\frac{3}{2}}}$.

    En conséquence : $\displaystyle (\frac{\pi }{2})^{2}=(\arcsin 1)^{2}=\underset{n=1}{\overset{+\infty }{\sum }}a_{n}$, soit : $\displaystyle \pi ^{2}=\underset{n=1}{\overset{+\infty }{\sum }}4a_{n}$.
    Pour une précision de $10^{-5}$ il faut $10^{12}$ termes. Mauvaise pioche : l'équivalent de $a_n$ le laissait prévoir. Pas étonnant, on s'est mis à l'extrémité de l'intervalle de convergence.

    Rentrons dedans : $\displaystyle (\frac{\pi }{6})^{2}=(\arcsin \frac{1}{2})^{2}=\underset{n=1}{\overset{+\infty }{\sum }}a_{n}(\frac{1}{2})^{2n}$, soit : $\displaystyle \pi ^{2}=\underset{n=1}{\overset{+\infty }{\sum }}36\frac{a_{n}}{4^{n}}$
    Pour une précision de $10^{-5}$ il faut $7$ termes. C'est déjà mieux.

    Ceci dort dans mes archives depuis un certain temps, je n'ai pas vérifié les calculs avant de vous les livrer.

    [*** modéré *** hors mathématiques. AD}

    Fr. Ch.
  • @Chaurien:

    Merci pour le calcul mais qui a une grave lacune.
    Tu supposes implicitement que $\zeta(2)=\frac{\pi^2}{6}$


    PS
    [*** modéré *** hors mathématiques. AD]

    PS2:
    Toujours avec l'utilisation du développement en série entière d'$arcsin^2 x$ mais sans rien supposer sur la valeur de $\zeta(2)$:
    https://www.maa.org/sites/default/files/Gerald_Kimble21622.pdf
  • @Chaurien : comment obtiens-tu ton équivalent ? merci.
  • @ FdP
    Ce n'est pas une lacune. Mon propos était de proposer un procédé de calcul de $\pi ^2$ (ou de $\frac{\pi^2}{6}$) avec un nombre raisonnable de décimales et un nombre raisonnable de termes à calculer, et non de prouver une fois de plus que $\zeta(2)=\frac{\pi^2}{6}$.
    Toi tu « sais » que $ \displaystyle \int_0^1\frac{\ln(1+x)}{x}\,dx=\frac{1}{2}\zeta(2)$. Eh bien moi je « sais » que $\zeta(2)=\frac{\pi^2}{6}$ ;-).
    Dans cet exemple, comme dans le tien, on voit que pour améliorer la précision d'un calcul il vaut mieux compter sur la qualité de l'algorithme plus que sur le nombre de termes à calculer. En effet l'augmentation du nombre de termes, si elle augmente la précision théorique, augmente aussi les erreurs d'arrondi, sur quoi on n'a que peu de prise.
    Bonne soirée.
    Fr. Ch.
  • @totem
    Tu es étudiant je présume ? Il me semble que dans toutes les prépas, et sans doute à la fac, on traite la formule de Wallis en exercice en première année. Autrefois on la traitait même en Terminale C... Cet équivalent en est un corollaire immédiat. Sinon, on peut exprimer $a_n$ avec des factorielles, et appliquer la formule de Stirling, mais c'est plus compliqué, et moins cohérent puisque la formule de Wallis sert justement à déterminer la constate qui intervient dans la formule de Stirling.
    Bonne soirée.
    Fr. Ch.
  • Chaurien:

    J'ai démontré que $\displaystyle \int_0^1 \frac{\ln(1+x)}{x}\,dx=-\frac{1}{2}\int_0^1\frac{\ln(1-x)}{x}\,dx=-\frac{1}{2}\zeta(2)$

    La dernière égalité n'étant pas difficile du tout à établir directement sans théorème d'"échange" intégrale-somme.


    PS:
    $\zeta(2)$ est défini par , $\displaystyle \zeta(2)=\sum_{n=1}^{\infty} \frac{1}{n^2}$
  • OK j'aurais du me douter que racine de Pi impliquait Stirling...

    Etudiant moi ?? j'ai 37 ans :-) mais quand on (re)fait des maths n'est-on pas un éternel étudiant toute sa vie finalement ?

    Sinon la méthode d'Euler pour trouver Zeta (2) est fantastique :

    https://fr.wikipedia.org/wiki/Probl%C3%A8me_de_B%C3%A2le#La_d.C3.A9monstration_d.27Euler
  • @FdP
    Oui ces intégrales sont bien connues, je les pose en colle à l'occasion. On peut démontrer ces égalités avec les théorèmes concernant les suites et séries de fonctions, ou parfois sans ces théorèmes, en intégrant une somme finie d'une suite géométrique, qui présente l'avantage d'avoir un reste explicite. Ce qui permet de poser ceci dans les prépas HEC ou BCPST.
    Au fait, quelle est la référence de l'article de Math. Magazine que tu nous as communiqué ? Merci. Cette revue est de plus en plus difficile à trouver.
    Bonne soirée.
    Fr. ch.
  • Chaurien:

    http://www.jstor.org/stable/2690408

    (le lien pour le PDF était sur Math exchange, un coup de Jstor et j'ai retrouvé les références de l'article)

    On peut consulter quelques articles de Jstor en ligne (sans pouvoir télécharger de pdf). C'est assez limité (3 articles autorisés à la consultation tous les quinze jours je crois si on s'inscrit gratuitement. Tous les articles ne sont pas consultables gratuitement. En général ce sont les plus anciens qui sont accessibles de cette façon).

    Totem:
    Son calcul et tous les autres qui relèvent de cette méthode (il a publié des quantités phénoménales de formules obtenues de cette façon je pense) ne sont pas compatibles avec les critères de rigueur d'une démonstration.
    Si Euler s'est fatigué à trouver deux autres méthodes je pense que même à son époque ses confrères mathématiciens avaient des objections sur sa "preuve".

    PS:
    LA référence sur l'analyse des travaux d'Euler à notre époque est le travail de Sandifer qui est en ligne:
    http://eulerarchive.maa.org/hedi/

    Le premier article de ce mathématicien sur le traitement du problème de Bâle par Euler, http://eulerarchive.maa.org/hedi/HEDI-2003-12.pdf
    Le deuxième (qui rejoint les derniers messages de ce fil), http://eulerarchive.maa.org/hedi/HEDI-2004-03.pdf

    Un calcul sur les logarithmes, http://eulerarchive.maa.org/hedi/HEDI-2005-07.pdf
    (l'article confirme ce dont je me doutais, à l'époque d'Euler on avait des tables de logarithmes)

    Sur l'irrationalité de e, http://eulerarchive.maa.org/hedi/HEDI-2006-02.pdf
    Sur les séries divergentes, http://eulerarchive.maa.org/hedi/HEDI-2006-06.pdf
    Sur le produit eulérien, http://eulerarchive.maa.org/hedi/HEDI-2006-07.pdf
    Sur la constante d'Euler, http://eulerarchive.maa.org/hedi/HEDI-2007-10.pdf
  • Précisions sur la série de terme général $\displaystyle a_{n}=\frac{2\cdot 4\cdot ...\cdot (2n-2)}{3\cdot 5\cdot ...\cdot (2n-1)}\cdot \frac{1}{n}$.

    Pour $n\in \mathbb{N}$, les intégrales de Wallis sont : $\displaystyle w_{n}=\int_{0}^{\frac{\pi }{2}}(\cos \theta)^{n}d\theta =\int_{0}^{\frac{\pi }{2}}(\sin \theta )^{n}d\theta $.

    Soit $\displaystyle p_{n}=\frac{1\cdot 3\cdot ...\cdot (2n-1)}{2\cdot 4\cdot ...\cdot (2n)}$ pour $n>0$, et $p_{0}=1$. Alors : $\displaystyle p_{n}=\frac{(2n)!}{2^{2n}(n!)^{2}}=\frac{1}{2^{2n}}(_{~n}^{2n})$.
    Ce nombre a une grande importance en analyse, en théorie des nombres et en calcul des probabilités.
    On a : $\displaystyle w_{2n}=p_{n}\frac{\pi }{2},w_{2n+1}=\frac{1}{(2n+1)p_{n}},w_{2n-1}=\frac{1}{%
    2np_{n}}$, et comme la suite $w_n$ est décroissante, on en déduit : $\displaystyle p_{n}\sim \frac{1}{\sqrt{\pi n}}$ quand $n\rightarrow +\infty $.
    Pour notre série de terme général $a_{n}$ il en résulte : $\displaystyle a_{n}=\frac{2\cdot 4\cdot ...\cdot (2n-2)}{3\cdot 5\cdot ...\cdot (2n-1)}\cdot \frac{1}{n}=\frac{1}{2n^{2}p_{n}}\sim \frac{\sqrt{\pi }}{2n\sqrt{n}}$.
    Ce qui implique pour le reste : $\displaystyle R_{n}=\underset{k=n+1}{\overset{+\infty }{\sum }}a_{n}\sim \underset{k=n+1}{\overset{+\infty }{\sum }}\frac{\sqrt{\pi }}{2k\sqrt{k}}\sim
    \int_{n}^{+\infty }\frac{\sqrt{\pi }}{2t\sqrt{t}}dt=\frac{\sqrt{\pi }}{\sqrt{n}}$.
    La convergence est vraiment très mauvaise, encore plus mauvaise que celle de la série des inverses carrés.
    Noter qu'au passage on a répondu à la question posée dans le fil
    http://www.les-mathematiques.net/phorum/read.php?4,1427668
    Bonne journée.
    Fr. Ch.
    16/08/2017
Connectez-vous ou Inscrivez-vous pour répondre.